Problems discussed in the blog post
STEMS Mathematics 2023 Category B P3
Let ABC be a triangle whose incircle \omega touches sides BC, CA, AB at D,E,F respectively. Let H be the orthocenter of DEF and let altitude DH intersect \omega again at P and EF intersect BC at L. Let the circumcircle of BPC intersect \omega again at X. Prove that points L,D,H,X are concyclic.
Proof: Define G as midpoint of LD, J as midpoint of FD, K as midpoint of ED. Note that G-J-K collinear.
Claim: BJKC cyclic
Proof: Note that\angle JKD=\angle FED
Note that (L,D;B,C)=-1. So GB\cdot GD=GD^2.
Claim: PXJK cyclic
Proof: Invert PBC about the incirlce, we get B\rightarrow J, C\rightarrow K.
Claim: G-X-P collinear
Proof: Radical axis on (DFE),(BJKC),(XPJK)
So GD^2=GB\cdot GC=GJ\cdot GK=GX\cdot GP.
So GD is tangent (XPD). So \angle XPD=\angle XDL.
To show XHDL cyclic, it is enough to show LH is tangent to (XPH).
Note that LP=LH as EF is perpendicular bisector of PH.
Now, we have the reduced problem.
Problem: Given ABC a triangle, define E=AA\cap BC. Define H as the orthocenter and D'=AH\cap (ABC). Define X=(AHE)\cap (ABC). Prove that EH is tangent to (HXD').
Proof:\angle EHX=\angle EAX=\angle ACX=\angle AD'X=\angle HD'X.
We are done~~
Brazilian Math Olympiad 2007, Problem 5
Let ABCD be a convex quadrangle, P the intersection of lines AB and CD, Q the intersection of lines AD and BC and O the intersection of diagonals AC and BD. Show that if \angle POQ= 90^\circ then PO is the bisector of \angle AOD and OQ is the bisector of \angle AOB.
Proof: Define QO\cap AB=X,PO\cap BC=Y. Note that by ceva-menalaus, we have (P,X;A,B)=-1=(B,C;F,E). Since \angle POX=90\implies \angle AOX=\angle XOB. By angle bisector harmonic lemma. Similarly, we have \angle BOF=\angle COF. Done!
USA TSTST 2016 Problem 2, by Evan Chen
Let ABC be a scalene triangle with orthocenter H and circumcenter O. Denote by M, N the midpoints of \overline{AH}, \overline{BC}. Suppose the circle \gamma with diameter \overline{AH} meets the circumcircle of ABC at G \neq A, and meets line AN at a point Q \neq A. The tangent to \gamma at G meets line OM at P. Show that the circumcircles of \triangle GNQ and \triangle MBC intersect at a point T on \overline{PN}.
Proof: Note that Q is the humpty point. So AG,HQ,BC concur at say X. Note that by brocards N-H-G collinear and HG\perp AG. So O-M-P||N-H-G. Also note that Q\in (GNX).
Claim: PGMA cyclic
Proof: Since M is centre of (GMH), we get\angle GMA=2\angle GHA=2\angle PGA=2(90-\angle GPM)=180-\angle GPA.
Claim: MP diameter of AGMP
Proof:\angle MAG=\angle MGA=90-\angle GMP= 90-\angle PAG.
Define Y=(APGM)\cap (BMC). By radicals on (MBC),(APGM),(ABCG) we get BC,AG,MY concurring at X.
Define E,F as feet of the altitude on AC,AB. Radicals on (AH),(FEBC),(ABC)\implies E-F-X Note that MYEF cyclic asXY\cdot XM=XB\cdot XC=XE\cdot XF.
Claim: P-Y-N collinear
Proof: As MN is the diameter of N_9 we get90=\angle MEN=\angle MYN.
So Y\in (XGQN). Hence Y is the desired concurrency.
Iranian TST 2020, second exam day 2, problem 4
Let ABC be an isosceles triangle (AB=AC) with incenter I. Circle \omega passes through C and I and is tangent to AI. \omega intersects AC and circumcircle of ABC at Q and D, respectively. Let M be the midpoint of AB and N be the midpoint of CQ. Prove that AD, MN and BC are concurrent.
Proof: Define X=AD\cap BC.
Claim: N is the centre of \omega.
Proof: Define N' such that N'I\perp AI and N'\in AC. Note that \angle N'CI=\angle BCI=\angle N'IC\implies N'\text{ is the center of }\omega\implies N'=N.
Define P=BC\cap \omega. Define P' as the reflection of P wrt N. Then note that \angle QP'C=90\implies P'\in \omega\implies QP'||BC.
Claim: P'\in AC
Proof: \angle AP'Q=\angle QPD=\angle 90-\angle DPX=\angle AXB\implies P'\in AD.
So by homothety, we get X-N-M collinear.
2022 IZHO P3
In parallelogram ABCD with acute angle A a point N is chosen on the segment AD, and a point M on the segment CN so that AB = BM = CM. Point K is the reflection of N in line MD. The line MK meets the segment AD at point L. Let P be the common point of the circumcircles of AMD and CNK such that A and P share the same side of the line MK. Prove that \angle CPM = \angle DPL.
Proof: We begin with the claim.
Claim: AMNK cyclic
Proof: Let \angle MAB=\theta\implies \angle ABM=180-2\theta. Let \angle MBC=\alpha \implies \angle ABC=180-2\theta+\alpha
So \angle MNK=\angle DNK-\angle DNC=\angle DNK-(180-\angle NCD-\angle NDC)
Moreover, \angle MAN=\angle BCD-\angle MAB=2\theta-\alpha-\theta=\theta-\alpha.
So \angle MAN=\angle MNK\implies AMNK\text{ is cyclic }.
Claim: MK||AB||BC
Proof: \angle MKD=\angle MND=\angle DNC=\alpha.
So \angle MKD=\angle KDC=\alpha\implies MK||AB||BC.
Define O as the circumcenter of (AMD).
Claim: O-L-M-K collinear
Proof: \angle NMO=\angle AMO-\angle AMN=\frac{180-\angle AOM}{2}-\angle AMN
Note that AB=BM=MC=MD. Note that OC is the perpendicular bisector of MOD.
Claim: CKNO cyclic
Proof: \angle KOC=\angle MAD=\angle MAN=\angle MKN=\angle MNK.
Infact, MO=MC=CD and NOCK is isosceles trapezoid.
Define (AMD)\cap (CNKO)=P,Q. We use radical axis on (CKNOP),(AKDO),(AMDP). We get PQ,OK,AD concurring at L.
We use radicals on (ANMK),(AMD),(NKC) we get NK,PQ,AM concurring at say X.
Claim: M is incenter of AKD
Proof: \angle AKM=\angle DNM=\angle MKD
But note that AMDP is appollinius circle i.e define M' as antipode of M in (AMD), we have (M',M;L,K)=-1. So \angle MPK=\angle MPQ.
To finish, note that \angle CPM=\angle CPK+\angle MPK
Some random problems I did
Problem 1: Let ABCD a convex quadrilateral. Suppose that the circumference with center B and radius BC is tangent to AD in F and the circumference with center A and radius AD is tangent to BC in E. Prove that DE and CF are perpendicular.
Proof: Note that BF\perp AF, AE\perp BE\implies AFEB \text{ is cyclic}.
Note that\angle FCE=\angle CFB=90-\frac{\angle FBE}{2}=90-\frac{\angle DAE}{2}=\angle FDE\implies DCEF\text{ is cyclic}.
Problem 2: Consider an acutangle triangle ABC with circumcenter O. A circumference that passes through B and O intersect sides BC and AB in points P and Q. Prove that the orthocenter of triangle OPQ is on AC.
Proof: Define R as the (AQO)\cap AC. We claim that R is the orthocenter of OPQ.
For RO\perp PQ, note that \angle OQP=90-A and A=\angle QAR=180-\angle QOR. So RO\perp PQ. For RP\perp QO, note that \angle ORP=\angle OCP=90-A and\angle QPR=\angle QPO+\angle RPQ=\angle OBA+\angle OCA=90-C+90-B\implies RP\perp QO.
Random configurations pics
We use a lot of stuffs in USATST 2013 P2. Firtly, we use prism lemma, which states like, if two lines l_1 and l_2 meet at say X and A_1,B_1,C_1 are points on l_1 and A_2,B_2,C_2 are points on l_2. We have A_1A_2,B_1B_2,C_1C_2 concur iff (A_1,B_1;C_1,X)=(A_2,B_2;C_2,X). Concurrency and prism lemma goes well with each other.
Comments
Post a Comment